Problem: proving that function is constant [closed]











up vote
4
down vote

favorite
1












Let $f colon mathbb{R} to mathbb{R} $ be a continuous function so that $ f(x) in mathbb I = mathbb Rsetminus mathbb Q, forall x in mathbb{R}$. Prove that $f$ is constant.



I tried assuming it's not a constant but I can't get to a contradiction with continuity.










share|cite|improve this question















closed as off-topic by user21820, Xander Henderson, Did, TheSimpliFire, Chinnapparaj R Nov 20 at 2:58


This question appears to be off-topic. The users who voted to close gave this specific reason:


  • "This question is missing context or other details: Please improve the question by providing additional context, which ideally includes your thoughts on the problem and any attempts you have made to solve it. This information helps others identify where you have difficulties and helps them write answers appropriate to your experience level." – user21820, Xander Henderson, Did, TheSimpliFire, Chinnapparaj R

If this question can be reworded to fit the rules in the help center, please edit the question.









  • 2




    What's $mathbb{I}$?
    – Dante Grevino
    Nov 19 at 12:26










  • @DanteGrevino it's set of irrational numbers
    – user560461
    Nov 19 at 12:27






  • 5




    Hint: assume it is not and then use the Intermediate Value Theorem to show that $f$ must take on a rational value.
    – John Douma
    Nov 19 at 12:29






  • 1




    Basically, rationals are everywhere. You cannot move an inch, a millimeter or an angstrom without stepping on a rational, or, for that matter, an infinity of rationals.
    – Eric Duminil
    Nov 19 at 15:50















up vote
4
down vote

favorite
1












Let $f colon mathbb{R} to mathbb{R} $ be a continuous function so that $ f(x) in mathbb I = mathbb Rsetminus mathbb Q, forall x in mathbb{R}$. Prove that $f$ is constant.



I tried assuming it's not a constant but I can't get to a contradiction with continuity.










share|cite|improve this question















closed as off-topic by user21820, Xander Henderson, Did, TheSimpliFire, Chinnapparaj R Nov 20 at 2:58


This question appears to be off-topic. The users who voted to close gave this specific reason:


  • "This question is missing context or other details: Please improve the question by providing additional context, which ideally includes your thoughts on the problem and any attempts you have made to solve it. This information helps others identify where you have difficulties and helps them write answers appropriate to your experience level." – user21820, Xander Henderson, Did, TheSimpliFire, Chinnapparaj R

If this question can be reworded to fit the rules in the help center, please edit the question.









  • 2




    What's $mathbb{I}$?
    – Dante Grevino
    Nov 19 at 12:26










  • @DanteGrevino it's set of irrational numbers
    – user560461
    Nov 19 at 12:27






  • 5




    Hint: assume it is not and then use the Intermediate Value Theorem to show that $f$ must take on a rational value.
    – John Douma
    Nov 19 at 12:29






  • 1




    Basically, rationals are everywhere. You cannot move an inch, a millimeter or an angstrom without stepping on a rational, or, for that matter, an infinity of rationals.
    – Eric Duminil
    Nov 19 at 15:50













up vote
4
down vote

favorite
1









up vote
4
down vote

favorite
1






1





Let $f colon mathbb{R} to mathbb{R} $ be a continuous function so that $ f(x) in mathbb I = mathbb Rsetminus mathbb Q, forall x in mathbb{R}$. Prove that $f$ is constant.



I tried assuming it's not a constant but I can't get to a contradiction with continuity.










share|cite|improve this question















Let $f colon mathbb{R} to mathbb{R} $ be a continuous function so that $ f(x) in mathbb I = mathbb Rsetminus mathbb Q, forall x in mathbb{R}$. Prove that $f$ is constant.



I tried assuming it's not a constant but I can't get to a contradiction with continuity.







functions continuity






share|cite|improve this question















share|cite|improve this question













share|cite|improve this question




share|cite|improve this question








edited Nov 20 at 0:14









Surb

37.1k94375




37.1k94375










asked Nov 19 at 12:24









user560461

525




525




closed as off-topic by user21820, Xander Henderson, Did, TheSimpliFire, Chinnapparaj R Nov 20 at 2:58


This question appears to be off-topic. The users who voted to close gave this specific reason:


  • "This question is missing context or other details: Please improve the question by providing additional context, which ideally includes your thoughts on the problem and any attempts you have made to solve it. This information helps others identify where you have difficulties and helps them write answers appropriate to your experience level." – user21820, Xander Henderson, Did, TheSimpliFire, Chinnapparaj R

If this question can be reworded to fit the rules in the help center, please edit the question.




closed as off-topic by user21820, Xander Henderson, Did, TheSimpliFire, Chinnapparaj R Nov 20 at 2:58


This question appears to be off-topic. The users who voted to close gave this specific reason:


  • "This question is missing context or other details: Please improve the question by providing additional context, which ideally includes your thoughts on the problem and any attempts you have made to solve it. This information helps others identify where you have difficulties and helps them write answers appropriate to your experience level." – user21820, Xander Henderson, Did, TheSimpliFire, Chinnapparaj R

If this question can be reworded to fit the rules in the help center, please edit the question.








  • 2




    What's $mathbb{I}$?
    – Dante Grevino
    Nov 19 at 12:26










  • @DanteGrevino it's set of irrational numbers
    – user560461
    Nov 19 at 12:27






  • 5




    Hint: assume it is not and then use the Intermediate Value Theorem to show that $f$ must take on a rational value.
    – John Douma
    Nov 19 at 12:29






  • 1




    Basically, rationals are everywhere. You cannot move an inch, a millimeter or an angstrom without stepping on a rational, or, for that matter, an infinity of rationals.
    – Eric Duminil
    Nov 19 at 15:50














  • 2




    What's $mathbb{I}$?
    – Dante Grevino
    Nov 19 at 12:26










  • @DanteGrevino it's set of irrational numbers
    – user560461
    Nov 19 at 12:27






  • 5




    Hint: assume it is not and then use the Intermediate Value Theorem to show that $f$ must take on a rational value.
    – John Douma
    Nov 19 at 12:29






  • 1




    Basically, rationals are everywhere. You cannot move an inch, a millimeter or an angstrom without stepping on a rational, or, for that matter, an infinity of rationals.
    – Eric Duminil
    Nov 19 at 15:50








2




2




What's $mathbb{I}$?
– Dante Grevino
Nov 19 at 12:26




What's $mathbb{I}$?
– Dante Grevino
Nov 19 at 12:26












@DanteGrevino it's set of irrational numbers
– user560461
Nov 19 at 12:27




@DanteGrevino it's set of irrational numbers
– user560461
Nov 19 at 12:27




5




5




Hint: assume it is not and then use the Intermediate Value Theorem to show that $f$ must take on a rational value.
– John Douma
Nov 19 at 12:29




Hint: assume it is not and then use the Intermediate Value Theorem to show that $f$ must take on a rational value.
– John Douma
Nov 19 at 12:29




1




1




Basically, rationals are everywhere. You cannot move an inch, a millimeter or an angstrom without stepping on a rational, or, for that matter, an infinity of rationals.
– Eric Duminil
Nov 19 at 15:50




Basically, rationals are everywhere. You cannot move an inch, a millimeter or an angstrom without stepping on a rational, or, for that matter, an infinity of rationals.
– Eric Duminil
Nov 19 at 15:50










4 Answers
4






active

oldest

votes

















up vote
10
down vote













Assuming we know 1. The Intermediate Value Theorem and 2. that between any two irrationals there is a rational, then the proof by contradiction should work fine.



If $f(a)<f(b)$ then there has to be a rational number $r$ with $f(a)<r<f(b).$ By IVT, there is a $c$ such that $f(c)=r$, contradiction.






share|cite|improve this answer




























    up vote
    9
    down vote













    The image $f(mathbb{R})$ is path-connected and the only non-empty path-connected subspaces of $mathbb{I}$ are the points. So $f$ is constant.






    share|cite|improve this answer




























      up vote
      2
      down vote













      The continuous image of a connected set is connected.



      The only connected sets in $mathbb Rsetminus mathbb Q$ are single points.



      Done.






      share|cite|improve this answer




























        up vote
        2
        down vote













        Suppose that $f$ is not constant. Then there are $i_1, i_2 in mathbb I$ such that $i_1 ne i_2$ and $i_1,i_2 in f(mathbb R)$. WLOG we can assume that $i_1 <i_2$. Now pick some rational number in $(i_1, i_2)$. The intermediate value theorem gives $r in f(mathbb R)$, a contradiction.






        share|cite|improve this answer






























          4 Answers
          4






          active

          oldest

          votes








          4 Answers
          4






          active

          oldest

          votes









          active

          oldest

          votes






          active

          oldest

          votes








          up vote
          10
          down vote













          Assuming we know 1. The Intermediate Value Theorem and 2. that between any two irrationals there is a rational, then the proof by contradiction should work fine.



          If $f(a)<f(b)$ then there has to be a rational number $r$ with $f(a)<r<f(b).$ By IVT, there is a $c$ such that $f(c)=r$, contradiction.






          share|cite|improve this answer

























            up vote
            10
            down vote













            Assuming we know 1. The Intermediate Value Theorem and 2. that between any two irrationals there is a rational, then the proof by contradiction should work fine.



            If $f(a)<f(b)$ then there has to be a rational number $r$ with $f(a)<r<f(b).$ By IVT, there is a $c$ such that $f(c)=r$, contradiction.






            share|cite|improve this answer























              up vote
              10
              down vote










              up vote
              10
              down vote









              Assuming we know 1. The Intermediate Value Theorem and 2. that between any two irrationals there is a rational, then the proof by contradiction should work fine.



              If $f(a)<f(b)$ then there has to be a rational number $r$ with $f(a)<r<f(b).$ By IVT, there is a $c$ such that $f(c)=r$, contradiction.






              share|cite|improve this answer












              Assuming we know 1. The Intermediate Value Theorem and 2. that between any two irrationals there is a rational, then the proof by contradiction should work fine.



              If $f(a)<f(b)$ then there has to be a rational number $r$ with $f(a)<r<f(b).$ By IVT, there is a $c$ such that $f(c)=r$, contradiction.







              share|cite|improve this answer












              share|cite|improve this answer



              share|cite|improve this answer










              answered Nov 19 at 12:31









              B. Goddard

              18.1k21340




              18.1k21340






















                  up vote
                  9
                  down vote













                  The image $f(mathbb{R})$ is path-connected and the only non-empty path-connected subspaces of $mathbb{I}$ are the points. So $f$ is constant.






                  share|cite|improve this answer

























                    up vote
                    9
                    down vote













                    The image $f(mathbb{R})$ is path-connected and the only non-empty path-connected subspaces of $mathbb{I}$ are the points. So $f$ is constant.






                    share|cite|improve this answer























                      up vote
                      9
                      down vote










                      up vote
                      9
                      down vote









                      The image $f(mathbb{R})$ is path-connected and the only non-empty path-connected subspaces of $mathbb{I}$ are the points. So $f$ is constant.






                      share|cite|improve this answer












                      The image $f(mathbb{R})$ is path-connected and the only non-empty path-connected subspaces of $mathbb{I}$ are the points. So $f$ is constant.







                      share|cite|improve this answer












                      share|cite|improve this answer



                      share|cite|improve this answer










                      answered Nov 19 at 12:34









                      Dante Grevino

                      5016




                      5016






















                          up vote
                          2
                          down vote













                          The continuous image of a connected set is connected.



                          The only connected sets in $mathbb Rsetminus mathbb Q$ are single points.



                          Done.






                          share|cite|improve this answer

























                            up vote
                            2
                            down vote













                            The continuous image of a connected set is connected.



                            The only connected sets in $mathbb Rsetminus mathbb Q$ are single points.



                            Done.






                            share|cite|improve this answer























                              up vote
                              2
                              down vote










                              up vote
                              2
                              down vote









                              The continuous image of a connected set is connected.



                              The only connected sets in $mathbb Rsetminus mathbb Q$ are single points.



                              Done.






                              share|cite|improve this answer












                              The continuous image of a connected set is connected.



                              The only connected sets in $mathbb Rsetminus mathbb Q$ are single points.



                              Done.







                              share|cite|improve this answer












                              share|cite|improve this answer



                              share|cite|improve this answer










                              answered Nov 19 at 18:06









                              MPW

                              29.6k11856




                              29.6k11856






















                                  up vote
                                  2
                                  down vote













                                  Suppose that $f$ is not constant. Then there are $i_1, i_2 in mathbb I$ such that $i_1 ne i_2$ and $i_1,i_2 in f(mathbb R)$. WLOG we can assume that $i_1 <i_2$. Now pick some rational number in $(i_1, i_2)$. The intermediate value theorem gives $r in f(mathbb R)$, a contradiction.






                                  share|cite|improve this answer



























                                    up vote
                                    2
                                    down vote













                                    Suppose that $f$ is not constant. Then there are $i_1, i_2 in mathbb I$ such that $i_1 ne i_2$ and $i_1,i_2 in f(mathbb R)$. WLOG we can assume that $i_1 <i_2$. Now pick some rational number in $(i_1, i_2)$. The intermediate value theorem gives $r in f(mathbb R)$, a contradiction.






                                    share|cite|improve this answer

























                                      up vote
                                      2
                                      down vote










                                      up vote
                                      2
                                      down vote









                                      Suppose that $f$ is not constant. Then there are $i_1, i_2 in mathbb I$ such that $i_1 ne i_2$ and $i_1,i_2 in f(mathbb R)$. WLOG we can assume that $i_1 <i_2$. Now pick some rational number in $(i_1, i_2)$. The intermediate value theorem gives $r in f(mathbb R)$, a contradiction.






                                      share|cite|improve this answer














                                      Suppose that $f$ is not constant. Then there are $i_1, i_2 in mathbb I$ such that $i_1 ne i_2$ and $i_1,i_2 in f(mathbb R)$. WLOG we can assume that $i_1 <i_2$. Now pick some rational number in $(i_1, i_2)$. The intermediate value theorem gives $r in f(mathbb R)$, a contradiction.







                                      share|cite|improve this answer














                                      share|cite|improve this answer



                                      share|cite|improve this answer








                                      edited Nov 19 at 18:19









                                      MPW

                                      29.6k11856




                                      29.6k11856










                                      answered Nov 19 at 12:31









                                      Fred

                                      42.4k1642




                                      42.4k1642















                                          Popular posts from this blog

                                          QoS: MAC-Priority for clients behind a repeater

                                          Ивакино (Тотемский район)

                                          Can't locate Autom4te/ChannelDefs.pm in @INC (when it definitely is there)